Vector calculus, + finding parametric equation

Click For Summary
The discussion centers on finding a set of parametric equations for a line that intersects a given plane and is perpendicular to another line. The user successfully determined the intersection point P(1,2,-1) by substituting the line's equations into the plane equation. They calculated the cross product of the normal vectors from the plane and line to derive the direction of the new line. The resulting parametric equations for the perpendicular line are x=1-5t, y=-2+3t, z=-1+4t. Overall, the solution appears correct, with a minor typo noted in the intersection point.
CaptainOfSmug
Messages
13
Reaction score
0

Homework Statement


The line L
L1: x=3+2t, y=2t, z=t
Intersects the plane x+3y-z=-4 at a point P. Find a set of parametric equations for the line in the same plane that goes through P and is perpendicular to L.

Homework Equations


cross-product
r=r0+t(vector) this is to get in parametric form typically

The Attempt at a Solution


Well let's just say first I'm having trouble visually what this question is asking (is there a program where I can graph this type of stuff in 3 space?)

substituting the values of "L" into the plane equation and then solved for "t" which was -1. I then plugged those values into the parametric equations of the line to get the point of intersection P(1,2,-1)

Here is where I get confused and visually a bit shaky. I decided to use the normal direction vector, I'll call it
"n1" from the plane=<1,3,-1> and "n2" from the line =<2,2,1>
I take the cross product with the resultant vector being <-5,3,4>
So I then use the parametric equation which is: L2 => x=1-5t, y=-2+3t, z=-1+4t

I'm think I'm done at this point because now I have to sets of parametric equations. To check my answer I set
L1 and L2 equal to each other, solved for the variables using elimination and found the intersection point to be the same as I found earlier.

I have no idea if I'm right or wrong, any tips would be helpful! Please don't just tell me the answer, I would prefr someone to point out a mistake and let me figure out the rest, after all, I'm taking math to actually grasp it :)

Thanks in advance!
Cheers
 
Physics news on Phys.org
CaptainOfSmug said:

Homework Statement


The line L
L1: x=3+2t, y=2t, z=t
Intersects the plane x+3y-z=-4 at a point P. Find a set of parametric equations for the line in the same plane that goes through P and is perpendicular to L.

Homework Equations


cross-product
r=r0+t(vector) this is to get in parametric form typically

The Attempt at a Solution


Well let's just say first I'm having trouble visually what this question is asking (is there a program where I can graph this type of stuff in 3 space?)

substituting the values of "L" into the plane equation and then solved for "t" which was -1. I then plugged those values into the parametric equations of the line to get the point of intersection P(1,2,-1)

Here is where I get confused and visually a bit shaky. I decided to use the normal direction vector, I'll call it
"n1" from the plane=<1,3,-1> and "n2" from the line =<2,2,1>
I take the cross product with the resultant vector being <-5,3,4>
So I then use the parametric equation which is: L2 => x=1-5t, y=-2+3t, z=-1+4t

I'm think I'm done at this point because now I have to sets of parametric equations. To check my answer I set
L1 and L2 equal to each other, solved for the variables using elimination and found the intersection point to be the same as I found earlier.

I have no idea if I'm right or wrong, any tips would be helpful! Please don't just tell me the answer, I would prefr someone to point out a mistake and let me figure out the rest, after all, I'm taking math to actually grasp it :)

Thanks in advance!
Cheers

The intersection point is <1,-2,-1> but that's just a typo. Looks ok otherwise.
 
Question: A clock's minute hand has length 4 and its hour hand has length 3. What is the distance between the tips at the moment when it is increasing most rapidly?(Putnam Exam Question) Answer: Making assumption that both the hands moves at constant angular velocities, the answer is ## \sqrt{7} .## But don't you think this assumption is somewhat doubtful and wrong?

Similar threads

  • · Replies 9 ·
Replies
9
Views
1K
  • · Replies 7 ·
Replies
7
Views
2K
  • · Replies 2 ·
Replies
2
Views
1K
Replies
3
Views
2K
Replies
6
Views
4K
  • · Replies 1 ·
Replies
1
Views
2K
  • · Replies 4 ·
Replies
4
Views
3K
  • · Replies 4 ·
Replies
4
Views
2K
Replies
2
Views
1K
  • · Replies 6 ·
Replies
6
Views
2K